0

Can anybody please explain the derivation of transformation of a frame rotating with constant angular velocity w.r.t. a stationary frame. It can't understand it from my book. The derivation started with assuming origins of both frames K (stationary) and Kp(rotating) are same. So the position vectors are equal i.e r=rp .Then for displacement vectors we can write

$$ dr=v_p\cdot dt + (d\phi \times r)$$

so by dividing dt I got

$$v=v_p + (\omega \times r)$$

then

$$dv = dv_p + (\omega \times dr)$$

up to there is everything ok.

I thought by dividing by $dt$ I will obtain acceleration. But that gives a different result. I don't know why?

Floris
  • 118,905
  • 2
    What have you done so far to understand it? means which part you dont understand? – Paul Jan 02 '15 at 03:41
  • 1
    What's so hard about sines and cosines? Isn't that shown in your book? – CuriousOne Jan 02 '15 at 04:06
  • You can find all the calculi at http://physics.stackexchange.com/questions/148944/rotation-systems-problem-interpreting-an-equation/149303#149303 – Sofia Jan 02 '15 at 17:52
  • I think that several answers at the link that Sofia gave could be helpful. Please look at them and see whether you still have any questions after that. – Floris Jan 02 '15 at 22:10

0 Answers0